Source Latex: Corrigé du devoir de mathématiques en Terminale générale, spécialité mathématiques


Limites de suites et fonctions

Fichier
Type: Corrigé de devoir
File type: Latex, tex (source)
Télécharger le document pdf compilé pdficon
Description
Devoir maison corrigé de mathématiques, spécialité mathématiques en terminale générale: suites, récurrence, limites
Niveau
Terminale générale, spécialité mathématiques
Table des matières
  • Calculs de dérivées
  • Démonstration par récurrence
  • Étude de fonction
  • Suite récurrente définie par une fonction
Mots clé
limite, suite, récurrence, suite récurrente, construction des premiers termes, spécialité mathématiques, terminale générale
Voir aussi:

Documentation sur LaTeX
lien vers la documentation Latex
Source LaTex icone

Source Latex de la correction du devoir

\documentclass[12pt,onecolumn,a4paper]{article}

\usepackage[french]{babel}
%\selectlanguage{francais}
\usepackage[utf8]{inputenc}
\usepackage{amsfonts}
\usepackage{amssymb}
\usepackage{amsmath}
\usepackage{calc}
\usepackage{enumerate}
\usepackage{array}
\usepackage{pst-all}
\usepackage{hyperref}
\hypersetup{
    pdfauthor={Yoann Morel},
    pdfsubject={Correction du devoir de mathématiques terminale S: calcul algébrique, dérivée, sens de variation, suite récurrence},
    pdftitle={Corrigé du devoir de mathématiques},
    pdfkeywords={calcul algébrique, dérivée, sens de variation, suite récurrence, devoir corrigé, Mathématiques, TS, terminale S}
}
\hypersetup{
    colorlinks = true,
    linkcolor = blue,
    anchorcolor = red,
    citecolor = blue,
    filecolor = red,
    urlcolor = red
}
\voffset=-1cm

% Raccourcis diverses:
\newcommand{\nwc}{\newcommand}
\nwc{\dsp}{\displaystyle}
\nwc{\bge}{\begin{equation}}\nwc{\ene}{\end{equation}}
\nwc{\bgar}{\begin{array}}\nwc{\enar}{\end{array}}
\nwc{\bgit}{\begin{itemize}}\nwc{\enit}{\end{itemize}}
\nwc{\bgen}{\begin{enumerate}}\nwc{\enen}{\end{enumerate}}

\nwc{\la}{\left\{}\nwc{\ra}{\right\}}
\nwc{\lp}{\left(}\nwc{\rp}{\right)}
\nwc{\lb}{\left[}\nwc{\rb}{\right]}

\nwc{\ul}{\underline}
\nwc{\tm}{\times}
\nwc{\V}{\overrightarrow}
\newcommand{\zb}{\mbox{$0\hspace{-0.67em}\mid$}}
\newcommand{\db}{\mbox{$\hspace{0.1em}|\hspace{-0.67em}\mid$}}
\newcommand{\ct}{\centerline}

\nwc{\bgsk}{\bigskip}
\nwc{\vsp}{\vspace{0.1cm}}
\nwc{\vspd}{\vspace{0.2cm}}
\nwc{\vspt}{\vspace{0.3cm}}
\nwc{\vspq}{\vspace{0.4cm}}

\def\N{{\rm I\kern-.1567em N}}
\def\D{{\rm I\kern-.1567em D}}
\def\R{{\rm I\kern-.1567em R}}
\def\C{{\rm C\kern-4.7pt
\vrule height 7.7pt width 0.4pt depth -0.5pt \phantom {.}}}
\def\Q{\mathbb{Q}}
\def\Z{{\sf Z\kern-4.5pt Z}}
\def\euro{\mbox{\raisebox{.25ex}{{\it =}}\hspace{-.5em}{\sf C}}}

\newcounter{nex}[section]\setcounter{nex}{0}
\newenvironment{EX}{%
\stepcounter{nex}
\bgsk{\noindent{{\bf Exercice }}\arabic{nex}}\hspace{0.5cm}
}{}
\nwc{\bgex}{\begin{EX}}\nwc{\enex}{\end{EX}}

\nwc{\bgfg}{\begin{figure}}\nwc{\enfg}{\end{figure}}
\nwc{\epsx}{\epsfxsize}\nwc{\epsy}{\epsfysize}
\nwc{\bgmp}{\begin{minipage}}\nwc{\enmp}{\end{minipage}}
\newenvironment{centerpage}{\vspace*{\fill}}{
	\protect\vspace*{\fill}}
\setlength{\columnsep}{30pt}	% default=10pt
\setlength{\columnseprule}{1pt}	% default=0pt (no line)
\setlength{\headsep}{0in}		% default=0.35in
\setlength{\parskip}{0ex}
\setlength{\parindent}{0mm}
\voffset=-1cm
\textheight=27.6cm
\textwidth=19.2cm
\topmargin=-.6cm
\headheight=-0.cm
\footskip=1.cm
\oddsidemargin=-1.5cm

\usepackage{fancyhdr}
\pagestyle{fancyplain}
\setlength{\headheight}{0cm}
\renewcommand{\headrulewidth}{0pt}
\renewcommand{\footrulewidth}{.1pt}
\lfoot{Y. Morel \href{https://xymaths.fr/Lycee/Terminale-generale-specialite-mathematiques/}{ xymaths - spé maths en terminale générale}}
\cfoot{}
\rfoot{Correction du devoir de mathématiques - \thepage/\pageref{LastPage}}
%%%%%%%%%%%%%%%%%%%%%%%%%%%%%%%%%%%%%%%%%%%%%%%%%%%%%%%%%%%%%%%%%
\begin{document}

%\vspace*{-3em}

\ct{\bf\LARGE{Correction du devoir de math\'ematiques}}

\bgex
%$f_1'(x)=\dfrac{3x^4-35x^8-3}{x^2}$ \quad ; \quad 
%$f_1'(x)=2x$  \quad ; \quad 
%$f_2'(x)=5\dfrac{-x^2+3}{\left( x^2+3\rp^2}$  \quad ; \quad 
$f_3'(x)=\dfrac{4x}{\left( x^2+1\rp^2}$ \quad ; \quad 
%
%\bigskip
%$g_1'(x)=3e^{3x+2}$  \quad ; \quad 
%$g_2'(x)=(1+x)e^x$ \quad ; \quad 
$g_3'(x)=\dfrac{-e^x}{\left( e^x+1\rp^2}$ \quad ; \quad 
$g_4'(x)=e^{x^2}\,\dfrac{2xe^x+2x-e^x}{\left( 2xe^x+2x^2-e^x-1\rp^2}$
\enex


\bigskip
\bgex
C'est le signe de la dérivée de f qui nous donne son sens de variation.
Ici $f$ est la somme d'une fonction affine (que l'on dérive facilement) et de 
$\dfrac{2}{x-2}=2\times\dfrac{1}{u(x)}$ avec $u(x)=x-2$ dont la dérivée est $u'(x)=1$. 

La dérivée de $\dfrac{1}{u}$ étant $-\dfrac{u'}{u^2}$, on trouve donc que 
$f'(x)=2-\dfrac{2}{(x-2)^2}$.

Pour pouvoir déterminer le signe de cette expression, on l'exprime sur un seul dénominateur:   
\[f'(x)=\dfrac{2(x-2)^2-2}{(x-2)^2}=\dfrac{2x^2-8x+6}{(x-2)^2}=2\dfrac{x^2-4x+3}{(x-2)^2}\]

Le numérateur est un trinôme du second degré dont le discriminant est 
$\Delta=(-4)^2-4\times1\times3=4=2^2>0$ et qui admet donc deux racines 
$x_1=\dfrac{-(-4)-\sqrt4}{2\times1}=1$ et $x_2=\dfrac{-(-4)+\sqrt4}{2\times1}=3$.

On obtient alors le signe du trinôme, et donc de la dérivée, et enfin le sens de variation de $f$ (on n'oublie pas non plus la valeur interdite\dots). 

\[\begin{tabular}{|c|lcccccccr|}\hline
$x$&$-\infty$&&1&&2&&3&&$+\infty$\\\hline
$x^2-4x+3$ &&$+$&0&$-$&$|$&$-$&0&$+$&\\\hline
$(x-2)^2$ &&$+$&$|$&$+$&0&$+$&$|$&$+$&\\\hline
$f'(x)$ &&$+$&0&$-$&&$-$&0&$+$&\\\hline
&&&2&&&$+\infty$&&&$+\infty$\\
$f$&&\Large{$\nearrow$}&&\Large{$\searrow$}&
\psline(0,-.6)(0,1.3)\psline(.07,-.6)(.07,1.3)&
\Large{$\searrow$}&&\Large{$\nearrow$}&\\
&$-\infty$&&&$-\infty$&&&9&&\\\hline
\end{tabular}\]

{\bf Limites de $f$:} 
En l'infini, on a 
$\dsp\lim_{x\to-\infty}\dfrac2{x-2}=\lim_{x\to+\infty}\dfrac2{x-2}=0$ 
et 
$\dsp\lim_{x\to-\infty}2x+1=-\infty$ 
et 
$\dsp\lim_{x\to+\infty}2x+1=+\infty$ . 

On obtient alors, par somme des limites, 
$\dsp\lim_{x\to-\infty}f(x)=-\infty$
et 
$\dsp\lim_{x\to+\infty}f(x)=+\infty$

\bigskip
Lorsque $x\to2$, on $\dsp\lim_{x\to2}2x+1=5$, 
et $\dsp\lim_{x\to2}x-2=0$. 

Le quotient $\dfrac2{x-2}$ tend vers l'infini, et il reste à déterminer son signe en séparant les cas $x<2\iff x-2<0$ et $x>2\iff x-2>0$: 

$\dsp\lim_{\scriptsize\bgar{l}x\to2\\x<2\enar}\dfrac2{x-2}=-\infty$ et donc 
$\dsp\lim_{\scriptsize\bgar{l}x\to2\\x<2\enar}f(x)=-\infty$. 

On trouve de m\^eme 
$\dsp\lim_{\scriptsize\bgar{l}x\to2\\x<2\enar}\dfrac2{x-2}=+\infty$ et donc 
$\dsp\lim_{\scriptsize\bgar{l}x\to2\\x<2\enar}f(x)=+\infty$. 

On en déduit de plus que la droite d'équation $x=2$ est une asymptote verticale à $\mathcal{C}_f$. 

\[\psset{xunit=1cm,yunit=.4cm,arrowsize=8pt}
\begin{pspicture*}(-5.8,-17)(5.8,22)
\psline{->}(-5.8,0)(5.8,0)
\psline{->}(0,-17)(0,22)
\multido{\i=-5+1}{11}{\psline(\i,.3)(\i,-.3)\rput(\i,-1){\i}}
\multido{\i=-15+5}{8}{\psline(-.1,\i)(.1,\i)\rput[r](-.1,\i){\i}}
\psplot{-3}{1.9}{2 x mul 1 add 2 x 2 sub div add}
\psplot{2.1}{5}{2 x mul 1 add 2 x 2 sub div add}
\psline[linecolor=blue](2,-17)(2,22)
\end{pspicture*}\]
\enex

\clearpage
\bgex $f(x) = \dfrac{2 + 3x}{4 + x}$ \hfill {\it (Bac S: métropole - La Réunion 13 septembre 2019)}
%Soit $f$ la fonction définie sur l'intervalle [0~;~4] par



\medskip

\textbf{Partie A}
\begin{enumerate}
\item 
$u_1 = f\left(u_0\right) = \dfrac{2 + 9}{4 + 3} = \dfrac{11}{7}$.
\item La fonction $f$ est définie et dérivable sur [0~;~4] et sur cet intervalle :

$f'(x) = \dfrac{3(4 + x) - 1(2 + 3x)}{(4 + x)^2} = \dfrac{12 + 3x - 2 - 3x}{(4 + x)^2} = \dfrac{10}{(4 + x)^2}$

Quotient de nombres positifs ce nombre dérivé est positif quel que soit $x$ dans l'intervalle [0~;~4]. La fonction $f$ est donc croissante sur [0~;~4].  
\item Démonstration par récurrence :

\emph{Initialisation}

On a d'après la première question : $1 \leqslant u_1 \leqslant u_0 \leqslant 3$ : l'encadrement est vrai au rang $0$ ;

\emph{Hérédité}

Supposons que pour $n \in \N$, \; $1 \leqslant u_{n+1} \leqslant u_n \leqslant 3$ ; par croissance de la fonction $f$ sur [0~;~4], on 

$f(1) \leqslant f\left(u_{n+1}\right) \leqslant f\left(u_{n}\right) \leqslant f(3)$ ou car $f(1) = \dfrac{5}{5} = 1$ et $f(3) = \dfrac{11}{7} \leqslant 3$, 

$1 \leqslant u_{n+2} \leqslant u_{n+1} \leqslant 3$ : la relation est donc vraie au rang $n + 1$.

\emph{Conclusion} : l'encadrement est vrai au rang $0$ et s'il est vrai à un rang quelconque $n$ il est vrai au rang suivant $n+1$ : d'après le principe de récurrence pour tout naturel $n$, \; $1 \leqslant u_{n+1} \leqslant u_n \leqslant 3$. 
\item 
  \begin{enumerate}[a)]
  \item D'après la question précédente la suite $\left(u_n\right)$ est décroissante,  minorée par $1$ : elle converge donc vers une limite $\ell \geqslant 1$.
\item De l'égalité $u_{n+1} = f\left(u_n \right) = \dfrac{2 + 3u_n}{4 + u_n}$ on en déduit par continuité de la fonction $f$ (puisque $f$ est dérivable) :
\[\ell = \dfrac{2 + 3\ell}{4 + \ell}.\]

\item On en déduit que $\ell(4 + \ell) = 2 + 3\ell \iff \ell^2 + \ell - 2 = 0$.

Or $\Delta = 1 + 4 \times 2 = 9 = 3^2$. Il y a deux solutions :

$\ell_1 = \dfrac{- 1 - 3}{2} = -2$ et  $\ell_2 = \dfrac{- 1 + 3}{2} = 1$.

Comme $\ell \in [1~;~3]$, la seule solution est $\ell_2 = 1$.
  \end{enumerate}
\end{enumerate}

\bigskip

\textbf{Partie B}
\begin{enumerate}
\item Voir l'annexe.

On peut conjecturer que la suite $\left(v_n\right)$ est croissante et qu'elle a pour limite 1.
\item  
  \begin{enumerate}[a)]
  \item 
$1 - v_{n+1} = 1 - \dfrac{2 + 3v_n}{4 + v_n} = \dfrac{4 + v_n - 2 - 3v_n}{4 + v_n}= \dfrac{2 - 2v_n}{4 + v_n} = \dfrac{2}{4 + v_n}\left(1 - v_n\right)$.
  \item	
    \emph{Initialisation} pour $n = 0$, \; $1 - v_0 = 0,9$ ; or $\left(\dfrac{1}{2}\right)^0 = 1$.
	
On a bien $0 \leqslant 1 - v_0 \leqslant \left(\dfrac{1}{2}\right)^0$.	

\emph{Hérédité} Supposons qu'au rang $n \in \N$ quelconque, on ait $1 - v_n \leqslant \left(\dfrac{1}{2}\right)^n$.

On a $1 - v_{n+1} = \dfrac{2}{4 + v_n}\left(1 - v_n \right)$, donc d'après l'hypothèse de récurrence :

$1 - v_{n+1}  \leqslant \dfrac{2}{4 + v_n} \times \left(\dfrac{1}{2} \right)^n$.

Or $0 \leqslant 1 - v_n \leqslant \left(\dfrac{1}{2} \right)^n \iff v_n \geqslant 1 - \left(\dfrac{1}{2} \right)^n \geqslant 0$ ; il suit que $4 + v_n \geqslant 4$, donc en prenant les inverses $0 \leqslant \dfrac{1}{4 + v_n} \leqslant \dfrac{1}{4}$.

On a donc $0 \leqslant 1 - v_{n+1} \leqslant 2 \times \dfrac{1}{4}\left(\dfrac{1}{2} \right)^n$, soit finalement :

$0 \leqslant 1 - v_{n+1} \leqslant \left(\dfrac{1}{2} \right)^{n+1}$ : l'encadrement est vrai au rang $n + 1$.

L'encadrement est vrai au rang $0$ et s'il est vrai à un rang $n$quelconque il est vrai au rang $n + 1$ : d'après le principe de récurrence :

quel que soit le naturel $n$, \; $0 \leqslant 1 - v_n \leqslant \left(\dfrac{1}{2}\right)^n$. 
 	\end{enumerate}
\item  %La suite $\left(v_n\right)$ converge-t-elle ? Si oui, préciser sa limite.
Comme $0< \dfrac{1}{2} < 1$, on sait que $\displaystyle\lim_{n \to + \infty} \left(\dfrac{1}{2}\right)^n = 0$, donc l'encadrement trouvé à la question précédente montre que la la limite de $1 - v_n = 0$, donc :

\[\displaystyle\lim_{n \to + \infty}  v_n = 1.\]
\end{enumerate}



\begin{center}
\textbf{\large Annexe}

\psset{unit=11cm,comma=true,arrowsize=2pt 3}
\begin{pspicture}(-0.1,-0.1)(1.15,1.1)
%\psgrid
\psaxes[linewidth=1.25pt,Dx=0.1,Dy=0.1](0,0)(0,0)(1.15,1.1)
\psline(1.1,1.1)
\psplot[plotpoints=2000,linewidth=1.25pt,linecolor=blue]{0}{1.15}{3 x mul 2 add x 4 add div}
\uput[ul](1.1,1.1){$D$}\uput[d](1.1,1.03){\blue $\mathcal{C}_f$}
\psline[ArrowInside=->](0.1,0)(0.1,0.561)(0.561,0.561)(0.561,0.8074)(0.8074,0.8074)(0.8074,0.92)(0.92,0.92)
\psline(0.561,0)(0.561,0.561)
\psline(0.8074,0.8074)(0.8074,0)
\psline(0.92,0)(0.92,0.92)
\uput[d](0.1,-0.05){$v_0 =0,1$}\uput[d](0.561,-0.05){$v_1=0,561$}\uput[d](0.8074,-0.05){$v_2=0,807$}\uput[d](0.92,-0.05){$v_3=0,92$}
\end{pspicture}
\end{center}
\enex



\label{LastPage}
\end{document}

Télécharger le fichier source Latex